Proof of Froda's Theorem (explanation)











up vote
0
down vote

favorite













Theorem: Let $f$ be a real valued function of real variable defined on open interval $(a,b)$ and let $f$ be monotonic. Then the set of all discontinuities is at most countable.




I would like an explanation for several steps in the proof given here



The proof:



WLOG let $f$ be monotonic increasing and let $x_0$ be a point of discontinuity, because $f$ is discontinuous at $x_0$, it follows that $lim_{xto x_0}f(x)neq f(x_0)$. Denote
$$f(x_0^-)=lim_{xto x_0^-}f(x)$$
$$f(x_0^+)=lim_{xto x_0 ^+}f(x)$$



Because $f$ is increasing, it follows that $f(x_0^-)<f(x_0^+)$ (Because $f$ is discontinuous, the strict ienquality holds.) Define intervals for each $x_i$ (the point of discontinuity)
$$S(x_i)={ymid f(x_i^-)<y<f(x_i^+)}$$
Now, the system of $S(x_i)$'s is pairwise disjoint and each of $S(x_i)$ is contained in compact interval $[f(a),f(b)]$. Now, it sufficies to pick a random rational number from each $S(x_i)$ thus associating each $x_i$ with some number $qinmathbb{Q}$ thus making an injection $psi:bigcup S(x_i)rightarrow mathbb{Q}$. Thus $bigcup S(x_i)$ is at most countable.



My questions:




  1. What about the case when the one sided limits would go to infinity?
    Is this a rigorous-enough explanation?


It is clear that if any left-sided limit goes to $+infty$ at some point, say $x_i$, then in the interval $(x_i,b)$ is atleast one more point, say $r_1$ for which the value $f(r_1)>infty$ (because $f$ is monotonic increasing), but that's not possible, since nothing is above $infty$.



Simillarly, if any left-sided limit goes to $-infty$ at some point, say $x_j$, then in the inrval $(a,x_i)$ there is some $r_2$ such that $f(r_2)<-infty$...etc.



Is my reasoning correct?










share|cite|improve this question






















  • Regarding the use of the name "Froda", see Brian S. Thomson's answer to Proof of Froda's theorem and my answer to A search for theorems which appear to have very few, if any hypotheses.
    – Dave L. Renfro
    Nov 15 at 6:55















up vote
0
down vote

favorite













Theorem: Let $f$ be a real valued function of real variable defined on open interval $(a,b)$ and let $f$ be monotonic. Then the set of all discontinuities is at most countable.




I would like an explanation for several steps in the proof given here



The proof:



WLOG let $f$ be monotonic increasing and let $x_0$ be a point of discontinuity, because $f$ is discontinuous at $x_0$, it follows that $lim_{xto x_0}f(x)neq f(x_0)$. Denote
$$f(x_0^-)=lim_{xto x_0^-}f(x)$$
$$f(x_0^+)=lim_{xto x_0 ^+}f(x)$$



Because $f$ is increasing, it follows that $f(x_0^-)<f(x_0^+)$ (Because $f$ is discontinuous, the strict ienquality holds.) Define intervals for each $x_i$ (the point of discontinuity)
$$S(x_i)={ymid f(x_i^-)<y<f(x_i^+)}$$
Now, the system of $S(x_i)$'s is pairwise disjoint and each of $S(x_i)$ is contained in compact interval $[f(a),f(b)]$. Now, it sufficies to pick a random rational number from each $S(x_i)$ thus associating each $x_i$ with some number $qinmathbb{Q}$ thus making an injection $psi:bigcup S(x_i)rightarrow mathbb{Q}$. Thus $bigcup S(x_i)$ is at most countable.



My questions:




  1. What about the case when the one sided limits would go to infinity?
    Is this a rigorous-enough explanation?


It is clear that if any left-sided limit goes to $+infty$ at some point, say $x_i$, then in the interval $(x_i,b)$ is atleast one more point, say $r_1$ for which the value $f(r_1)>infty$ (because $f$ is monotonic increasing), but that's not possible, since nothing is above $infty$.



Simillarly, if any left-sided limit goes to $-infty$ at some point, say $x_j$, then in the inrval $(a,x_i)$ there is some $r_2$ such that $f(r_2)<-infty$...etc.



Is my reasoning correct?










share|cite|improve this question






















  • Regarding the use of the name "Froda", see Brian S. Thomson's answer to Proof of Froda's theorem and my answer to A search for theorems which appear to have very few, if any hypotheses.
    – Dave L. Renfro
    Nov 15 at 6:55













up vote
0
down vote

favorite









up vote
0
down vote

favorite












Theorem: Let $f$ be a real valued function of real variable defined on open interval $(a,b)$ and let $f$ be monotonic. Then the set of all discontinuities is at most countable.




I would like an explanation for several steps in the proof given here



The proof:



WLOG let $f$ be monotonic increasing and let $x_0$ be a point of discontinuity, because $f$ is discontinuous at $x_0$, it follows that $lim_{xto x_0}f(x)neq f(x_0)$. Denote
$$f(x_0^-)=lim_{xto x_0^-}f(x)$$
$$f(x_0^+)=lim_{xto x_0 ^+}f(x)$$



Because $f$ is increasing, it follows that $f(x_0^-)<f(x_0^+)$ (Because $f$ is discontinuous, the strict ienquality holds.) Define intervals for each $x_i$ (the point of discontinuity)
$$S(x_i)={ymid f(x_i^-)<y<f(x_i^+)}$$
Now, the system of $S(x_i)$'s is pairwise disjoint and each of $S(x_i)$ is contained in compact interval $[f(a),f(b)]$. Now, it sufficies to pick a random rational number from each $S(x_i)$ thus associating each $x_i$ with some number $qinmathbb{Q}$ thus making an injection $psi:bigcup S(x_i)rightarrow mathbb{Q}$. Thus $bigcup S(x_i)$ is at most countable.



My questions:




  1. What about the case when the one sided limits would go to infinity?
    Is this a rigorous-enough explanation?


It is clear that if any left-sided limit goes to $+infty$ at some point, say $x_i$, then in the interval $(x_i,b)$ is atleast one more point, say $r_1$ for which the value $f(r_1)>infty$ (because $f$ is monotonic increasing), but that's not possible, since nothing is above $infty$.



Simillarly, if any left-sided limit goes to $-infty$ at some point, say $x_j$, then in the inrval $(a,x_i)$ there is some $r_2$ such that $f(r_2)<-infty$...etc.



Is my reasoning correct?










share|cite|improve this question














Theorem: Let $f$ be a real valued function of real variable defined on open interval $(a,b)$ and let $f$ be monotonic. Then the set of all discontinuities is at most countable.




I would like an explanation for several steps in the proof given here



The proof:



WLOG let $f$ be monotonic increasing and let $x_0$ be a point of discontinuity, because $f$ is discontinuous at $x_0$, it follows that $lim_{xto x_0}f(x)neq f(x_0)$. Denote
$$f(x_0^-)=lim_{xto x_0^-}f(x)$$
$$f(x_0^+)=lim_{xto x_0 ^+}f(x)$$



Because $f$ is increasing, it follows that $f(x_0^-)<f(x_0^+)$ (Because $f$ is discontinuous, the strict ienquality holds.) Define intervals for each $x_i$ (the point of discontinuity)
$$S(x_i)={ymid f(x_i^-)<y<f(x_i^+)}$$
Now, the system of $S(x_i)$'s is pairwise disjoint and each of $S(x_i)$ is contained in compact interval $[f(a),f(b)]$. Now, it sufficies to pick a random rational number from each $S(x_i)$ thus associating each $x_i$ with some number $qinmathbb{Q}$ thus making an injection $psi:bigcup S(x_i)rightarrow mathbb{Q}$. Thus $bigcup S(x_i)$ is at most countable.



My questions:




  1. What about the case when the one sided limits would go to infinity?
    Is this a rigorous-enough explanation?


It is clear that if any left-sided limit goes to $+infty$ at some point, say $x_i$, then in the interval $(x_i,b)$ is atleast one more point, say $r_1$ for which the value $f(r_1)>infty$ (because $f$ is monotonic increasing), but that's not possible, since nothing is above $infty$.



Simillarly, if any left-sided limit goes to $-infty$ at some point, say $x_j$, then in the inrval $(a,x_i)$ there is some $r_2$ such that $f(r_2)<-infty$...etc.



Is my reasoning correct?







real-analysis continuity proof-explanation monotone-functions






share|cite|improve this question













share|cite|improve this question











share|cite|improve this question




share|cite|improve this question










asked Nov 14 at 23:28









Michal Dvořák

831415




831415












  • Regarding the use of the name "Froda", see Brian S. Thomson's answer to Proof of Froda's theorem and my answer to A search for theorems which appear to have very few, if any hypotheses.
    – Dave L. Renfro
    Nov 15 at 6:55


















  • Regarding the use of the name "Froda", see Brian S. Thomson's answer to Proof of Froda's theorem and my answer to A search for theorems which appear to have very few, if any hypotheses.
    – Dave L. Renfro
    Nov 15 at 6:55
















Regarding the use of the name "Froda", see Brian S. Thomson's answer to Proof of Froda's theorem and my answer to A search for theorems which appear to have very few, if any hypotheses.
– Dave L. Renfro
Nov 15 at 6:55




Regarding the use of the name "Froda", see Brian S. Thomson's answer to Proof of Froda's theorem and my answer to A search for theorems which appear to have very few, if any hypotheses.
– Dave L. Renfro
Nov 15 at 6:55










1 Answer
1






active

oldest

votes

















up vote
1
down vote



accepted










$f$ is given to be real valued. There is no way $f(x-)$ or $f(x+)$ can be $infty$ or $-infty$ at any point. For example $f(y)leq f(x)$ if $a<y<x$ so $f(x-) leq f(x)<infty$ . Similarly, $f(x-) geq f(y)>-infty$ for any $y in (a,x)$, etc. Your argument for countability of the set of discontinuity points is correct.






share|cite|improve this answer





















    Your Answer





    StackExchange.ifUsing("editor", function () {
    return StackExchange.using("mathjaxEditing", function () {
    StackExchange.MarkdownEditor.creationCallbacks.add(function (editor, postfix) {
    StackExchange.mathjaxEditing.prepareWmdForMathJax(editor, postfix, [["$", "$"], ["\\(","\\)"]]);
    });
    });
    }, "mathjax-editing");

    StackExchange.ready(function() {
    var channelOptions = {
    tags: "".split(" "),
    id: "69"
    };
    initTagRenderer("".split(" "), "".split(" "), channelOptions);

    StackExchange.using("externalEditor", function() {
    // Have to fire editor after snippets, if snippets enabled
    if (StackExchange.settings.snippets.snippetsEnabled) {
    StackExchange.using("snippets", function() {
    createEditor();
    });
    }
    else {
    createEditor();
    }
    });

    function createEditor() {
    StackExchange.prepareEditor({
    heartbeatType: 'answer',
    convertImagesToLinks: true,
    noModals: true,
    showLowRepImageUploadWarning: true,
    reputationToPostImages: 10,
    bindNavPrevention: true,
    postfix: "",
    imageUploader: {
    brandingHtml: "Powered by u003ca class="icon-imgur-white" href="https://imgur.com/"u003eu003c/au003e",
    contentPolicyHtml: "User contributions licensed under u003ca href="https://creativecommons.org/licenses/by-sa/3.0/"u003ecc by-sa 3.0 with attribution requiredu003c/au003e u003ca href="https://stackoverflow.com/legal/content-policy"u003e(content policy)u003c/au003e",
    allowUrls: true
    },
    noCode: true, onDemand: true,
    discardSelector: ".discard-answer"
    ,immediatelyShowMarkdownHelp:true
    });


    }
    });














     

    draft saved


    draft discarded


















    StackExchange.ready(
    function () {
    StackExchange.openid.initPostLogin('.new-post-login', 'https%3a%2f%2fmath.stackexchange.com%2fquestions%2f2998964%2fproof-of-frodas-theorem-explanation%23new-answer', 'question_page');
    }
    );

    Post as a guest















    Required, but never shown

























    1 Answer
    1






    active

    oldest

    votes








    1 Answer
    1






    active

    oldest

    votes









    active

    oldest

    votes






    active

    oldest

    votes








    up vote
    1
    down vote



    accepted










    $f$ is given to be real valued. There is no way $f(x-)$ or $f(x+)$ can be $infty$ or $-infty$ at any point. For example $f(y)leq f(x)$ if $a<y<x$ so $f(x-) leq f(x)<infty$ . Similarly, $f(x-) geq f(y)>-infty$ for any $y in (a,x)$, etc. Your argument for countability of the set of discontinuity points is correct.






    share|cite|improve this answer

























      up vote
      1
      down vote



      accepted










      $f$ is given to be real valued. There is no way $f(x-)$ or $f(x+)$ can be $infty$ or $-infty$ at any point. For example $f(y)leq f(x)$ if $a<y<x$ so $f(x-) leq f(x)<infty$ . Similarly, $f(x-) geq f(y)>-infty$ for any $y in (a,x)$, etc. Your argument for countability of the set of discontinuity points is correct.






      share|cite|improve this answer























        up vote
        1
        down vote



        accepted







        up vote
        1
        down vote



        accepted






        $f$ is given to be real valued. There is no way $f(x-)$ or $f(x+)$ can be $infty$ or $-infty$ at any point. For example $f(y)leq f(x)$ if $a<y<x$ so $f(x-) leq f(x)<infty$ . Similarly, $f(x-) geq f(y)>-infty$ for any $y in (a,x)$, etc. Your argument for countability of the set of discontinuity points is correct.






        share|cite|improve this answer












        $f$ is given to be real valued. There is no way $f(x-)$ or $f(x+)$ can be $infty$ or $-infty$ at any point. For example $f(y)leq f(x)$ if $a<y<x$ so $f(x-) leq f(x)<infty$ . Similarly, $f(x-) geq f(y)>-infty$ for any $y in (a,x)$, etc. Your argument for countability of the set of discontinuity points is correct.







        share|cite|improve this answer












        share|cite|improve this answer



        share|cite|improve this answer










        answered Nov 14 at 23:46









        Kavi Rama Murthy

        41.3k31751




        41.3k31751






























             

            draft saved


            draft discarded



















































             


            draft saved


            draft discarded














            StackExchange.ready(
            function () {
            StackExchange.openid.initPostLogin('.new-post-login', 'https%3a%2f%2fmath.stackexchange.com%2fquestions%2f2998964%2fproof-of-frodas-theorem-explanation%23new-answer', 'question_page');
            }
            );

            Post as a guest















            Required, but never shown





















































            Required, but never shown














            Required, but never shown












            Required, but never shown







            Required, but never shown

































            Required, but never shown














            Required, but never shown












            Required, but never shown







            Required, but never shown







            Popular posts from this blog

            How do I know what Microsoft account the skydrive app is syncing to?

            When does type information flow backwards in C++?

            Grease: Live!